Você está na página 1de 14

Topology Homework # 8

1. Sutherland Page 147, Exercise 14.3. Prove that a closed subset of a


sequentially compact metric space is sequentially compact.
Proof. Let C be a closed subset of the sequentially compact metric space
X. If C = it is closed trivially, so assume C = and let {x
n
} be a
sequence in C. Since {x
n
} is also a sequence in X and since X is sequen-
tially compact, there is subsequence {x
n
k
} of {x
n
} converging to x X.
This means that x is a limit point of the sequence {x
n
k
}, which is a se-
quence in C because its a subsequence of {x
n
}. Since C is closed we must
have x C. Since {x
n
} was an arbitrary sequence in C with a convergent
subsequence whose limit is in C, it follows that C is sequentially compact.
1
Topology Homework # 8
2. Sutherland Page 147, Exercise 14.4 Prove that a sequentially com-
pact subspace of a metric space X is closed in X.
Proof. Let C be a sequentially compact subspace of a metric space X,
and let p X be a limit point of C. Since p is a limit point of C there is a
sequence {x
n
} in C such that x
n
p as n . Since C is sequentially
compact, there is a subsequence {x
n
k
} of {x
n
} converging to a point c C.
But x
n
p as n implies x
n
k
p as k , because subsequences
of convergent sequences converge to the same limit in metric spaces. Since
limits of sequences are unique in metric spaces it follows that c = p. Since
c C we have p C. Since p was an arbitrary limit point of C, we have
shown that C contains all its limit points; i.e. C is closed (see Proposition
6.17).
2
Topology Homework # 8
3. Sutherland Page 148, Exercise 14.7. Prove that any continuous map
from a sequentially compact metric space to another metric space has
bounded image.
Proof. Suppose X is a sequentially compact metric space and f : X Y
is continuous. By way of contradiction, suppose that f is not bounded.
Construct a sequence of points {x
n
} in X in the following manner: choose
x
1
X. Using the fact that f is not bounded, select x
2
X such that
d
Y
(f(x
1
), f(x
2
)) > 2. In general use the fact that f is not bounded to
select x
n
X such that d
Y
(f(x
1
), f(x
n
)) > n.
Now {x
n
} is a sequence in the sequentially compact metric space X, so
it has a convergent subsequence; say x
n
k
x X as k . By conti-
nuity of f, we then have f(x
n
k
) f(x) in Y as k . Since {f(x
n
k
)}
converges in Y , {f(x
n
k
)} is bounded in Y . But for all m N we have
d
Y
(f(x
n
k
), f(x
1
)) d
Y
(f(x
n
k
), f(x
n
m
)) +d
Y
(f(x
n
m
), f(x
1
)),
which implies
d
Y
(f(x
n
k
), f(x
1
))d
Y
(f(x
n
m
), f(x
1
)) d
Y
(f(x
n
k
), f(x
n
m
)) m N.
By construction of {x
n
} we have d
Y
(f(x
n
k
), f(x
1
)) as k . This
fact, together with the previous inequality, tells us that for each m N
d
Y
(f(x
n
k
), f(x
n
m
)) can be made arbitrarily large. Hence {f(x
n
k
)} is not
Cauchy in Y. But this immediately contradicts {f(x
n
k
)} converging to
f(x) in Y, because convergent sequences in metric spaces are necessarily
Cauchy. This contradiction tells us that f must be a bounded function.
3
Topology Homework # 8
4. Sutherland Page 148, Exercise 14.9. Prove that the product of two
sequentially compact metric spaces is sequentially compact.
Proof. Let X and Y be sequentially compact metric spaces and let
{(x
n
, y
n
)} be a sequence in X Y. Then {x
n
} = {p
X
(x
n
, y
n
)} is a se-
quence in X. Since X is sequentially compact, there is a subsequence
{x
n
k
} of {x
n
} converging to x X.
Now consider the subsequence {(x
n
k
, y
n
k
)} of {(x
n
, y
n
)} in X Y. Then
{y
n
k
} = {p
Y
(x
n
k
, y
n
k
)} is a sequence in Y . Since Y is sequentially com-
pact, there is a subsequence {y
n
k
j
} of {y
n
k
} converging to y Y.
Now consider the subsequence {(x
n
k
j
, y
n
k
j
)} of {(x
n
, y
n
)} in XY. Since
{x
n
k
j
} is a subsequence of {x
n
k
} in X and x
n
k
x as k , we have
x
n
k
j
x as j because subsequences of convergent sequences con-
verge to the same limit as the parent sequence. We now have x
n
k
j
x
and y
n
k
j
y as j . It follows that (x
n
k
j
, y
n
k
j
) (x, y) as j .
Now {(x
n
k
j
, y
n
k
j
)} is a convergent subsequence of {(x
n
, y
n
)} in X Y.
Since {(x
n
, y
n
)} was arbitrary in X Y, it follows that X Y is sequen-
tially compact whenever X and Y are sequentially compact.
4
Topology Homework # 8
5. Sutherland Page 148, Exercise 14.12. Prove that a subspace C of a
metric space X is relatively compact in X if and only if every sequence in
C has a convergent subsequence.
Proof.
() Suppose C is relatively compact in X and let {x
n
} be a sequence
in C. Since C C, {x
n
} is also a sequence in C. Since C is compact
(via relative compactness assumption), we know C is sequentially com-
pact. Hence {x
n
} (thought of as a sequence in C) admits a convergent
subsequence {x
n
k
} with limit x C. The subsequence {x
n
k
} is precisely
the convergent subsequence we desire.
() Suppose every sequence in C has a convergent subsequence, and let
{x
n
} be a sequence in C. Since x
n
C for each n N, there is, for each
n N, y
n
C such that d(x
n
, y
n
) < 1/n. Then {y
n
} is a sequence in
C. By assumption, there is a subsequence {y
n
k
} of {y
n
} such that {y
n
k
}
converges in X, say with limit y X. Since {y
n
k
} is also a sequence in C
with limit y, it follows that y C because the closure of C contains all
limit points of C.
Claim: x
n
k
y C.
Proof of Claim. Let > 0 be given. Choose N
1
N such that
d(y
n
k
, y) < /2 (1)
for all k N
1
. Now choose N
2
N such that 1/N
2
< /2. Let N =
max{N
1
, N
2
} Then, for all k N, we have
d(x
n
k
, y) d(x
n
k
, y
n
k
) +d(y
n
k
, y)
< d(x
n
k
, y
n
k
) +

2
(Via (1))
<
1
n
k
+

2
(Choice of y
n
s)

1
k
+

2
(k n
k
)

1
N
+

2
<

2
+

2
(N N
2
)
= .
Since > 0 was arbitrary, {x
n
k
} converges to y C. Since {x
n
} was an
arbitrary sequence in C, it follows that C is sequentially compact. Hence,
C is compact. By denition C is relatively compact in X.
5
Topology Homework # 8
6. Sutherland Page 148, Exercise 14.13. Give another proof that [a, b]
is connected along the following lines. Suppose {A, B} is a partition of
[a, b], where a A. Since A, B are open in [a, b], for each x [a, b]
there is some B
(x)
(x) entirely contained in either A or B. Let be a
Lebesgue number for the open cover {B
(x)
(x) : x [a, b]} of [a, b] and let
a = a
0
< a
1
< . . . < a
n
= b be such that a
i
a
i1
< for i = 1, . . . , n.
Deduce that [a, b] A.
Proof. We essentially follow the outline given in the problem statement.
Suppose {A, B} is a partition of [a, b]. Since A and B are open, cover
[a, b] and are disjoint (denition of partition), for each x [a, b] there is
(x) > 0 such that B
(x)
(x) is contained entirely in A or entirely in B.
Note that C = {B
(x)
(x) : x [a, b]} is an open cover of [a, b]. Since
[a, b] is compact it is sequentially compact; Proposition 14.18 (page 145)
then tells us that [a, b] has a Lebesgue number, say .
Choose N N large enough so that
ba
N
< . Let a
i
= a + i
ba
N
for
i = 0, . . . , N. Then S = {a
i
: i = 0, . . . , N} is a set of N +1 points in [a, b]
such that
a = a
0
< . . . < a
N
= b,
and for all i = 0, . . . , N 1
a
i+1
a
i
=

a + (i + 1)
b a
N

a +i
b a
N

=
b a
N
< ()
Furthermore, S forms an -net for [a, b]. Indeed, note that
[a, b] =
N1

i=0
[a
i
, a
i+1
].
So, if x [a, b], then x [a
i
x
, a
i
x
+1
] for some i
x
= 0, . . . , N 1. It follows
from () that |a
i
x
x| < . Since S is an -net for [a, b] we have
[a, b]
N

i=0
B

(a
i
) (2)
Since A and B cover [a, b], we have a A or a B. Without loss of
generality say a A (else relabel A and B). Since is the Lebesgue
number for C, we have B

(a) B
(x
a
)
(x
a
) for some x
a
[a, b]. By con-
struction B
(x
a
)
(x
a
) A or B
(x
a
)
(x
a
) B. Since A B = and
a B
(x
a
)
(x
a
)A we must have B
(x
a
)
(x
a
) A. Since B

(a) B
(x
a
)
(x
a
)
6
Topology Homework # 8
we have B

(a) A. Since a
1
a
0
< (see ()) we have a
1
B

(a), and
it follows from B

(a) B
(x
a
)
(x
a
) A that a
1
A.
Repeating the exact same argument just given proves B

(a
1
) A and a
2

A. Continuing inductively we obtain B

(a
i
) A for each i = 0, . . . , N.
Hence,
N

i=0
B

(a
i
) A.
Using the last line together with (2) we obtain
[a, b] A.
But [a, b] A implies [a, b] B = . This contradicts the assumption
that {A, B} is a partition for [a, b]. We must, therefore, have that [a, b] is
connected.
7
Topology Homework # 8
7. Sutherland Page 149, Exercise 14.17.
(a) Let X be a compact metric space with metric d and suppose that
f : X X satises d(f(x), f(y)) = d(x, y) for all x, y X. Prove
that f is onto (so f is an isometry).
(b) Let X, Y be compact metric spaces with metrics d
X
, d
Y
. Suppose
that the maps f : X Y and g : Y X satisfy d
Y
(f(x
1
), f(x
2
)) =
d
X
(x
1
, x
2
) and d
X
(g(y
1
), g(y
2
)) = d
Y
(y
1
, y
2
) for all x
1
, x
2
X and
all y
1
, y
2
Y. Prove that f and g are both onto (and hence isome-
tries).
(c) Construct a function f : (0, ) (0, ) which is not onto but
nevertheless satises |f(x) f(y)| = |x y| for all x, y (0, ).
Proof.
(a) Let x X. To prove that f is onto we must show x f(X). Dene
a sequence inductively in X as follows: x = x
1
and x
n+1
= f(x
n
).
Since f : X X, {x
n
} is a sequence in X. Now {x
n
} contains
a convergent subsequence because X is sequentially compact (by as-
sumption); lets denote the convergent subsequence by {x
n
k
}.
We proceed by using {x
n
k
} to prove that every -ball about x con-
tains a point from {x
n
}. Indeed, let > 0 be given. Since {x
n
k
} is
convergent it is necessarily Cauchy. Hence, there is K N such that
d(x
n
k
, x
n
m
) < m > k K (3)
In particular (3) holds for K and so we have
d(x
n
K
, x
n
m
) < m > K (4)
Using the assumption d(f(x), f(y)) = d(x, y) - call this assumption
() - iteratively n
K
1 times, we have for all m > K
d(x
n
K
, x
n
m
) = d(f(x
n
K
1
), f(x
n
m
1
))
= d(x
n
K
1
, x
n
m
1
) (Using ())
.
.
. (Finish iterating)
= d(x
1
, x
n
m
n
K
+1
)
= d(x, x
n
m
n
K
+1
)
We have established d(x
n
K
, x
n
m
) = d(x, x
n
m
n
K
+1
) for all m > K.
This deduction together with (4) implies
d(x, x
n
m
n
K
+1
) < m > K (5)
8
Topology Homework # 8
Since > 0 was arbitrary, it follows from (5) that every -ball about
x contains a point of {x
n
}. In other words, there is a subsequence
of {x
n
} converging to x. By construction of {x
n
}, x
n
f(X) for all
n 2. Hence, the subsequence converging to x is really a sequence
in f(X) converging to x. Thus, x f(X).
Now X is a sequentially compact metric space, so we know it is
compact. Since f is continuous and X is compact, it follows that
f(X) is compact. Since f(X) is a compact metric (thus Hausdor)
space, it is necessarily closed. Hence x f(X) = f(X); i.e. x f(X)
and so f is onto.
Note that d(f(x), f(y)) = d(x, y) for all x, y X implies f is one-to-
one. Indeed, suppose f(x) = f(y). Then 0 = d(f(x), f(y)) = d(x, y).
Since d(x, y) = 0 we have x = y, so f is one-to-one. Hence, f is a bi-
jective map with the property d(f(x), f(y)) = d(x, y) for all x, y X.
By denition, f is an isometry.
(b) Consider the map g f : X X. Using the assumptions of distance
preservation, observe that for all x, y X
d
X
((g f)(x), (g f)(y)) = d
X
(g(f(x)), g(f(y))
= d
Y
(f(x), f(y))
= d
X
(x, y)
Since X is compact and d
X
((g f)(x), (g f)(y)) = d
X
(x, y) for all
x, y X, it follows from part (a) that g f is onto. Since g f is
onto, g is necessarily onto. Using the same argument at the end of
part (a), g is therefore an isometry.
Reversing the roles of f and g above, as well as interchanging X
and Y in the appropriate places, we also obtain f g : Y Y is
onto. Since f g is onto, f is necessarily onto. Using the same
argument at the end of part (a), f is therefore an isometry.
(c) Consider the map f(x) = x + 1 dened on (0, ). This is does not
map (0, ) onto itself because x +1 > 1 for all x (0, ). However,
f does preserve distances. Indeed, note that if x, y (0, ), then
|f(x) f(y)| = |(x + 1) (y + 1)| = |x y|.
9
Topology Homework # 8
8. Sutherland Page 171, Exercise 15.6. Dene an equivalence relation
on [0, 1] by: x y if and only if either both x, y are rational or both are
irrational. Check that this is an equivalence relation, and prove that the
corresponding quotient space is the two-point space with the indiscrete
topology. (Note that [0, 1] is Hausdor but [0, 1]/ is not Hausdor.)
Proof.
We have x x for either x is rational or x is irrational; in either case
x, x are both rational or both irrational.
Now suppose x y. Then either both x, y are rational or both are
irrational. It follows that either both y, x are rational or both are
irrational (changing the order in which two numbers are written does
not magically change whether theyre rational or not). Hence y x.
Suppose x y and y x. Then either x, y both are rational or both
are irrational and either both y, z are rational or both are irrational.
Now if both x and y are rational, then, since y z, z must be rational
and so x z. If both x and y are irrational, then, since y z, z
must be irrational and so x z. In either case we have x z.
The above proves that is reexive, symmetric and transitive. Thus,
is an equivalence relation.
We now note that every point in R, and therefore [0, 1], is either rational
or irrational and so [0, 1]/ is a two-point space. Let [0, 1]/ = {[x], [y]}
where [x] is the equivalence class of the rationals in [0, 1] and [y] is the
equivalence class of the irrationals in [0, 1], and let p : [0, 1] [0, 1]/
be the natural map. To see that the quotient topology on [0, 1]/ is the
indiscrete topology observe that
p
1
([x]) = [0, 1] Q & p
1
([y]) = Irrationals in [0, 1].
Then neither p
1
([x]) nor p
1
([y]) is open in [0, 1], because every open
subset of [0, 1] contains both rational and irrational points. Since p
1
([x])
contains no irrationals and p
1
([y]) contains no rationals they cannot be
open in [0, 1], as claimed. Now p
1
([0, 1]/ ) = [0, 1], which is open in
[0, 1]. Hence, by denition of the quotient topology, the only open subsets
of [0, 1]/ are and [0, 1]/ . Therefore [0, 1]/ has the indiscrete
topology, as claimed.
10
Topology Homework # 8
9. Sutherland Page 172, Exercise 15.7. Prove the following necessary
and sucient condition for a map f : X Y from a space X onto a space
Y to be a quotient map: a subset V of Y is closed in Y if and only if
f
1
(V ) is closed in X.
Proof. We will repeatedly use Proposition 3.8 to justify f
1
(Y V ) =
X f
1
(V ) below.
() Suppose f : X Y is a quotient map.
Suppose V is closed in Y . Since V is closed in Y , Y V is open
in Y. Since f is a quotient map and Y V is open in Y we have
f
1
(Y V ) = X f
1
(V ) is open in X. Since X f
1
(V ) is open
in X, f
1
(V ) is closed in X, as desired.
Suppose f
1
(V ) is closed in X. We then have Xf
1
(V ) = f
1
(Y
V ) is open in X. Since f is a quotient map and f
1
(Y V ) is open
in X, Y V is open in Y . Hence, V is closed in Y.
()
Suppose a subset V of Y is closed in Y if and only if f
1
(V ) is closed in
X.
Suppose U Y is open in Y. Then Y U is closed in Y. By assump-
tion, f
1
(Y U) = X f
1
(V ) is closed in X. Hence, f
1
(V ) is
open in X.
Suppose f
1
(V ) is open in X. Then X f
1
(V ) = f
1
(Y V ) is
closed in X. By assumption, Y V is closed in Y. Hence, V is open
in Y.
Since f was assumed onto at the onset, the last two items prove that f is
a quotient map.
11
Topology Homework # 8
10. Sutherland Page 172, Exercise 15.8. Prove that the composition of
two quotient maps is again a quotient map.
Proof. Suppose f : X Y and g : Y Z are quotient maps. Then, by
denition of quotient map, f and g are both onto. It follows that g f
is also onto. Indeed, if z Z, then there is y Y such that g(y) = z
because g is onto. Since f is onto there is x X such that f(x) = y. We
then have g f(x) = z, and g f is onto, as claimed.
Now suppose U Z is open in Z. Since g is a quotient map, g
1
(U)
is open in Y. Since g
1
(U) is open in Y and f is a quotient map we have
f
1
(g
1
(U)) is open in X; i.e. (g f)
1
(U) is open in X.
Now suppose (g f)
1
(U) is open in X. Since (g f)
1
(U) = f
1
(g
1
(U))
is open in X and f is a quotient map we have g
1
(U) is open in Y. Since
g
1
(U) is open in Y and g is a quotient map we have U is open in Z.
The previous two paragraphs together with the observation that g f
is onto tell us that g f : X Z is a quotient map.
12
Topology Homework # 8
11. Sutherland Page 180, Exercise 16.2. Given that (f
n
), (g
n
) converge
to functions f, g uniformly on D, prove that (f
n
+ g
n
) converges to
f +g uniformly on D for any , R.
Proof. Let > 0 be given. Since (f
n
) converges to f uniformly and
(g
n
) converges to g uniformly, there are N
1
, N
2
N such that
|f(x) f
n
(x)| < n N
1
, x D
and
|g(x) g
n
(x)| < n N
2
, x D.
Let N = max{N
1
, N
2
}. Then for all n N and x D we have
|(f +g)(x) (f
n
+g
n
)(x)| = |f(x) f
n
(x) +g(x) g
n
(x)|
|f(x) f
n
(x)| +|g(x) g
n
(x)|
< +
= ( +)
Let K = +. Then, for all x D, we have
|(f +g)(x) (f
n
+g
n
)(x)| < K n N.
Since > 0 was arbitrary, Lemma 4.14 (page 23) implies (f
n
+ g
n
)
converges to f +g uniformly.
13
Topology Homework # 8
12. Sutherland Page 180, Exercise 16.4. Suppose that (f
n
) converges to
a function f uniformly on D. Prove that if each f
n
is bounded on D, then
(a) f is bounded on D;
(b) there is a uniform bound for the f
n
, i.e. there is K R such that
|f
n
(x)| K for all x D and all n N.
Proof.
(a) Let = 1. Since (f
n
) converges uniformly to f, there is N N such
that
|f(x) f
N
(x)| < 1 x D (6)
Since f
N
is bounded (by assumption), there is M
N
N such that
|f
N
(x)| < M
N
x D (7)
Then, for all x D we have
|f(x)| = |f(x) f
N
(x) +f
N
(x)|
|f(x) f
N
(x)| +|f
N
(x)|
< 1 +M
N
(Using (6) & (7))
Since the |f(x)| < 1 +M
N
holds for all x D, f is bounded on D.
(b) Let = 1. Since (f
n
) converges to f uniformly, there is N N such
that
|f
n
(x) f(x)| < 1 n N, x D (8)
From part (a) we know f is bounded by some number, call it K.
Then, for all n N we have
|f
n
(x)| = |f
n
(x) f(x) +f(x)|
|f
n
(x) f(x)| +|f(x)|
< 1 +|f(x)| (Via (8))
1 +K
Hence,
|f
n
(x)| < 1 +K n N, x D (9)
Since each f
n
is bounded (by assumption), there are M
1
, . . . , M
N1

N such that, for each i = 1, . . . , N 1,
|f
i
(x)| M
i
x D (10)
Let M = max{1 + K, M
1
, . . . , M
N1
}. It then follows from (9) and
(10) that for all n N
|f
n
(x)| M x D.
Hence, (f
n
) is uniformly bounded.
14

Você também pode gostar